11. Übung
Bearbeitung vom 17.01.07 - 22.01.07
Aufgabe 1: Fehlerfortpflanzung
Wir schreiben die Formel um nach

\begin{displaymath}
g = 4 \pi^{2} \frac{L}{T^{2}} = 4 \pi^{2} \frac{99.8}{2.03^{2}} cm \; s^{-2} = 956.09 \; cm \; s^{-2}
\end{displaymath}

Die partiellen Ableitungen sind
$\displaystyle \frac{\partial g}{\partial L}$ $\textstyle =$ $\displaystyle \frac{4\pi^{2}}{T^{2}} = 9.58 \; s^{-2}$  
$\displaystyle \frac{\partial g}{\partial T}$ $\textstyle =$ $\displaystyle - \frac{8\pi^{2} L}{T^{3}} = - 942 \; cm \; s^{-3}$  

Der quadratische Fehler ist dann

\begin{displaymath}
(\Delta g)^{2} = \left( \frac{\partial g}{\partial L} \right...
...tial T}
\right)^{2} (\Delta T)^{2} = 2226 \; cm^{2} \; s^{-4}.
\end{displaymath}

Wir verwenden bei der Angabe nur 2 Stellen des Fehlers, daher $\Delta g = 47 \; cm \; s^{-2}$. Unser gemessenes Ergebnis ist also $g = (9.56 \pm 0.47) \; m \; s^{-2}$.
Aufgabe 2: Gewichtetes Mittel
a) Aus

\begin{displaymath}
\chi^{2} = \sum_{i} \frac{(x_{i} - \overline{x})^{2}}{(\Delta x_{i})^{2}} =! \; Minimum
\end{displaymath}

folgt

\begin{displaymath}
\frac{\partial \chi^{2}}{\partial \overline{x}} = -2 \sum_{i} \frac{x_{i} - \overline{x}}{(\Delta x_{i})^{2}} = 0
\end{displaymath}

Mit den Gewichten $w_{i} = 1/(\Delta x_{i})^{2}$ also

\begin{displaymath}
\sum_{i} w_{i} x_{i} - \overline{x} \sum_{i} w_{i} = 0
\end{displaymath}

oder

\begin{displaymath}
\overline{x} = \frac{\sum_{i} w_{i} x_{i}}{\sum_{i} w_{i}}
\end{displaymath}

b) Das Programm Zmass.C bestimmt diese Mittelwerte. Der offizielle Wert in der PDG Tabelle ist allerdings nur aus den letzten 5 Ergebnissen bestimmt, liegt also etwas höher als der von uns errechnete Wert.
Aufgabe 3: Maximum Likelihood
a) Wir setzen die Formel für die Poisson- Verteilung

\begin{displaymath}
f(k; \lambda) = e^{-\lambda} \frac{\lambda^{k}}{k!}
\end{displaymath}

in die Likelihood- Formel ein:

\begin{displaymath}
L -2 \sum_{i=1}^{N} ln f(k_{i}, \lambda) = -2 \sum_{i=1}^{N} [(-\lambda) + k_{i} ln(\lambda) - ln(k_{i}!)]
\end{displaymath}

Die Ableitung nach $\lambda$ liefert:

\begin{displaymath}
\frac{\partial L}{\partial \lambda} = - 2 \sum_{i=1}^{N} [(-1) + \frac{k_{i}}{\lambda}] = 0
\end{displaymath}

Daraus folgt

\begin{displaymath}
-N + \frac{1}{\lambda} \sum_{i=1}^{N} k_{i} = 0
\end{displaymath}

und

\begin{displaymath}
\lambda = \frac{1}{N} \sum_{i=1}^{N} k_{i}
\end{displaymath}

b) Im Programm poisson.C bestimmen wir zunächst den Mittelwert. Es ergibt sich zu 2.4. Theoretisch sollte der symmetrische Fehler durch $\sqrt{\overline{n}/N} \approx 0.155$ gegeben sein. Aus der Maximum Likelihood Kurve lesen wir einen Wert von $0.15$ ab. Das Endergebnis ist also: $\lambda = 2.40 \pm 0.15$.



Harm Fesefeldt
2007-01-16